A rectangular dog pen is constructed using a barn wall as one side and 120m of fencing for the other three sides. a) give a sketch of the situation. b) express the area in terms of x. c) sketch the graph. d) find the value of x that gives the greatest area. e) find the dimensions of the pen. f) state the domain.

Answers

Answer 1

Answer:

see the attachments for the sketch and the graphA = x(120 -2x)x = 30 maximizes the areadimensions: 30 m by 60 mdomain of x: [0, 60]

Step-by-step explanation:

You want an expression for area, maximum area, and dimensions of a rectangular pen fenced on three sides with a total of 120 m of fencing.

a) Sketch

The first attachment shows a sketch of the pen with x defined as the dimension the pen extends from the barn wall. The total length of fence is 120 m, so the side parallel to the barn will be (120 -2x) m in length.

b) Area

The area is the product of the length and width of the rectangle:

  A = (x)(120 -2x)

c) Graph

The second attachment shows a graph of the area as a function of x.

d) Greatest area

The value of x that gives the greatest area is the x-coordinate of the vertex of the parabola. It is halfway between the zeros at x=0 and x=60. The maximum area will be had when x=30.

e) Dimensions

The dimensions of the pen are ...

  x = 30

  120 -2x = 120 -2(30) = 60

The pen is 30 m by 60 m, with the 60 m dimension parallel to the barn wall.

f) Domain

The area function only makes sense for values of x between 0 and 60 meters.

The domain of x is 0 ≤ x ≤ 60.

A Rectangular Dog Pen Is Constructed Using A Barn Wall As One Side And 120m Of Fencing For The Other
A Rectangular Dog Pen Is Constructed Using A Barn Wall As One Side And 120m Of Fencing For The Other

Related Questions

At the clothing store where you work, T-shirts are on sale, 3 for $10. A customer at the store says that one T-shirt should cost $7 at this rate. What error did the customer make?

The customer divided 10 by 3 instead of dividing 3 by 10.
The customer divided 3 by 10 instead of dividing 10 by 3.
The customer subtracted 3 from 10 instead of dividing 10 by 3.
The customer added 10 and 3 instead of dividing 3 by 10.
The customer subtracted 3 from 10 instead of dividing 3 by 10.

Answers

The error that the customer made was that he subtracted 3 from 10 instead of dividing 3 by 10.

It is given in the question that at the clothing store T-shirts are on sale, 3 for $10. A customer at the store says that one T-shirt should cost $7 at this rate.

We have to find the error made by the customer while calculating the rate of a T- shirt.

To find the rate of a shirt we will divide the number of shirts by the total amount.

Hence, the rate of a shirt will be $  10/3

But the customer subtracted 3 from 10 to get $ 7 as the rate of a T-shirt.

Hence, the customer subtracted 3 from 10 instead of dividing 3 by 10.

To learn more about divide, here:-

https://brainly.com/question/15381501

#SPJ1

Simply the expression
3x 3/648x4y8

Answers

Answer:

8%3-29xy

Step-by-step explanation:

thats the answer

If f(x)=√x-3, which inequality can be used to find the domain of f(x)?
square root of x - 3 over or equal to zero, x minus three over or equal to zero, square root of x-3 under or equal to zero; x-3 under or equal to 0

Answers

The inequality that can be used to find the domain of f(x) is x - 3 ≥ 0.

The correct answer is the second one i.e.x minus three over or equal to zero.

What is the Domain of a Square Root Function?Fundamentally, the square root function has the form f(x) = √x. In other words, f(x) = √x is the parent square root function.A negative number's square root is not a real number. Therefore, negative values cannot be put into the square root function.That is, the set of all real numbers that are not negative constitutes the domain of the square root function, f(x) = √x.Consequently, [0,∞ ) is the square root function domain. It should be noted that the domain also contains 0.

Given:

Function f(x) = √x-3

The fact that a negative number cannot be used as the term in the root square, so

x - 3 ≥ 0  which implies that, x ≥ 3.

From the inequality, we can tell that the domain of the function is the interval [3,∞).

Hence the inequality is x - 3 ≥ 0.

To learn more about Domain of Square Root Function visit:

brainly.com/question/26684809

#SPJ9

In ΔLMN, \overline{LN} LN is extended through point N to point O, \text{m}\angle MNO (5x+2)^{\circ}m∠MNO(5x+2)∘,\text{m}\angle NLM = (x+10)^{\circ}m∠NLM=(x+10) ∘,and \text{m}\angle LMN = (x+16)^{\circ}m∠LMN=(x+16)∘. What is the value of x?x?

Answers

Using the exterior angle theorem, the value of x in the triangle is: x = 8.

What is the Exterior Angle Theorem?

The exterior angle theorem states that the measure of the angle formed outside a triangle when a side is extended through one of the points of the triangle is equal to the sum of the remote interior angles of the triangle.

Triangle LMN is shown in the diagram attached below, where:

The measure of angle MNO = (5x + 2)°

The measure of angle NLM = (x + 10)°

The measure of angle LMN = (x + 16)°

Therefore:

Measure of angle MNO = measure of angle NLM + measure of angle LMN [based on the exterior angle theorem]

Substitute

(5x + 2)° = (x + 10)° +  (x + 16)°

Solve for x

5x + 2 = x + 10 +  x + 16

Combine like terms

5x + 2 = 2x + 26

5x + 2 - 2x = 2x - 2x + 26 [subtraction property of equality]

3x + 2 = 26

3x + 2 - 2 = 26 - 2  [subtraction property of equality]

3x = 24

3x/3 = 24/3 [division property of equality]

x = 8

Learn more about exterior angle theorem on:

https://brainly.com/question/11356657

#SPJ1

Hello I need help for my study guide :(

Answers

A rough sketch of the rectangular plot is

The perimeter of the rectangle is given by the formula:

[tex]P=2l+2w[/tex]

Where,

and,

i

The perimeter is 320, so we can write:

[tex]\begin{gathered} P=2l+2w \\ 2l+2w=320 \\ 2(l+w)=320 \\ l+w=160 \end{gathered}[/tex]

Total fencing cost $3120 and this is 1 length side and 2 width side.

The fencing on length side is $24 per foot and width side is $4 per foot.

e can write an equation:

[tex]\begin{gathered} 24l+2(4w)=3120 \\ 24l+8w=3120 \end{gathered}[/tex]

Now, we have to solve the 2 simultaneous equations,

[tex]\begin{gathered} l+w=160 \\ 24l+8w=3120 \end{gathered}[/tex]

Let's solve with elimination >>>>

[tex]\begin{gathered} l+w=160 \\ 24l+8w=3120 \\ ----------- \\ -8\times(l+w=160) \\ 24l+8w=3120 \\ ------------ \\ -8l-8w=-1280 \\ 24l+8w=3120 \\ _{\ldots\ldots\ldots\ldots\ldots\ldots\ldots\ldots\ldots\ldots\ldots\ldots\text{.}} \\ 16l=1840 \\ l=\frac{1840}{16} \\ l=115 \end{gathered}[/tex]

We can substitute this value of "l" into the first equation and solve for "w". This is shown below:

[tex]\begin{gathered} l+w=160 \\ 115+w=160 \\ w=160-115 \\ w=45 \end{gathered}[/tex]

Thus,

AnswerThe Longer Dimension is 115 feetThe Shorter Dimension is 45 feet

1) Which of the following is the additive inverse of a - b ?

Answers

     The number that produces zero when multiplied by an is known as the additive inverse of a number, or a, in mathematics. The opposite (number),[1] sign change,[2] and negation are other names for this number. [3] The additive inverse (opposite number) of a positive number is negative, and the additive inverse of a negative number is positive, when applied to a real number. The additive inverse of zero is zero.

Associated with subtraction

   Subtraction, which can be seen as adding the opposite, is closely related to additive inverse:

   a − b  =  a + (−b).

On the other hand, additive inverse can be compared as subtracting from zero:

−a = 0 − a.

As a result, the unary minus sign can be used to represent subtraction without using the "0" symbol, even though there shouldn't be a space after the unary "-" in proper typography.

Other features

 Negation has the following algebraic characteristics in addition to the identities mentioned above:

It is an Involution operation when (a) = a.

−(a + b) = (−a) + (−b)

−(a − b) = b − a

a − (−b) = a + b \s(−a) × b = a × (−b) = −(a × b)

(−a) × (−b) = a × b

Particularly, (a)2 = a2.

To Learn more About Additive inverse, Refer:

https://brainly.com/question/1548537

#SPJ13

Marcel and his 8 friends have a snow shoveling business. Last season, they earned a total of $2,240.64. How much
money did each friend make?
$280.08
B$17,925.12
$248.96
D) $20,165.76
int(s)
HIDE HINT
1/1
| How many people were there all together?

Answers

the answer is $248.96
there are 9 people dividing the money amongst them
please vote brainliest:)

Two distinct lines, & and m, are each perpendicular to the same line n.
a. What is the measure of the angle where line / meets line n?
b. What is the measure of the angle where line m meets line n?
A. The measure of the angle where line | meets line n is ___ degrees
B. The measure of the angle where line meets line n is ____ degrees

Answers

The two distinct lines, line l and line m, have the following relationship with line n

A. The measure of the angle where line | meets line n is 90 degrees

B. The measure of the angle where line meets line n is 90 degrees

What are perpendicular lines?

This is a term used to refer to lines that intersect at angle 90 degrees. perpendicular lines are also found in right angle triangle. Hence it is right to say that two sides of a right angled triangle are perpendicular to each other.

The features of perpendicularity also helps in defining a squire and rectangles. This is true because the sides meet at angle 90 degrees

Read more on perpendicular lines here: https://brainly.com/question/1202004

#SPJ1

There are 18 boys and 46 girls in class. 87.5% of the students in the class take the
bus to school. How many students do not take the bus to school?

Answers

Answer:  12.5%

Reason:

87.5% of the students take the bus to school

100% - 87.5% = 12.5% of the students do not take the bus to school

The info about the "18 boys and 46 girls" is put there as a distraction since it's never used.

Equation IQ include many puzzles to advance in the game. 1 puzzle involves a pair of similar triangles. Players must use the clues to find missing side lengths, two triangles are given.

Answers

Since the two triangles in the given are similar triangles, we now know that they both have the same angle, but scaled, meaning they have different side lengths.

Base on the given, XA = 15, while XZ = 27

We can now assume that if XA : XZ, BA : YZ

We can rewite this as:

[tex]\frac{XA}{XZ}=\frac{BA}{YZ}[/tex]

Now, plugging in the values of XA, XZ, and BA, we can now solve for YZ.

[tex]\frac{XA}{XZ}=\frac{BA}{YZ}[/tex][tex]\frac{15}{27}=\frac{20}{YZ}[/tex][tex]15YZ=20(27)[/tex][tex]YZ=\frac{20(27)}{15}[/tex][tex]YZ=36[/tex]

We now see that the length of YZ is 36 inches.

Next, to solve for XY, we will use the same method, but this time we will be using XB instead of BA.

[tex]\frac{XA}{XZ}=\frac{XB}{XY}[/tex]

[tex]\frac{15}{27}=\frac{10}{XY}[/tex][tex]15XY=10(27)[/tex][tex]XY=\frac{10(27)}{15}[/tex][tex]XY=18[/tex]

Therefore, XY is 18 inches long.

13. A journalist believes that, in reality, these components have a shorter lifespan than what the company is reporting. He tests a random group of 35 of these components and finds a mean lifespan of 4.05 years. a. What is the mean of the sampling distribution of sample means when 35 random components are tested?

Answers

The mean of the sampling distribution of sample means when 35 random components are tested will be 4.05 years.

How to illustrate the information?

From the information, it should be noted that the journalist believes that, in reality, these components have a shorter lifespan than what the company is reporting.

He then tests a random group of 35 of these components and finds a mean lifespan of 4.05 years.

Therefore from the information, we acne see that the mean lifespan has already been given.

Therefore, the mean lifespan is 4.05 years.

Learn more about mean on:

brainly.com/question/1136789

#SPJ1

if all tree friends worked 20 hours during the week, how much ith each person earn

Answers

The amount of money earned by each person will be;

Savannah = $190

Greg = $180

Kevin = $165

How to calculate the amount?

From the information, the following can be deduced:

Savannah works $9.50 per hour.

Greg works $9 per hour.

Kevin works $8.25 per hour

Therefore, they all work for 20 hours. The amount for each person will be:

Savannah works $9.50 per hour. = $9.50 × 20 = $190

Greg works $9 per hour. = $9 × 20 = $180

Kevin works $8.25 per hour = $8.25 × 20 = $165

Learn more about money on:

brainly.com/question/24373500

#SPJ1

Savannah works $9.50 per hour.

Greg works $9 per hour.

Kevin works $8.25 per hour

If all tree friends worked 20 hours during the week, how much does each person earn?

Graph the function h(x) =-3x-1

Answers

we have the function

h(x)=-3x-1

this is the equation of a line

To graph a line we need at least two points

so

Find out the intercepts

step 1

Find out the x-intercept (value of x when the value of y=0)

For h(x)=0

substitute

0=-3x-1

3x=-1

x=-1/3

the x-intercept is (-1/3,0)

as the value is a fraction, take another point

For x=2

h(x)=-3(2)-1

h(x)=-7

we have the point (2,-7)

step 2

Find out the y-intercept (value of h(x) when the value of x=0)

For x=0

h(x)=-3(0)-1

h(x)=-1

the y-intercept is (0,-1)

step 3

we have the points

(2,-7)

(0,-1)

To graph, the line, plot the given points and join them

using a graphing tool

Can you please explain this to me?

Answers

The amplitude is determined as the distance from the midline of the cosine function.

From the graph provided, the depth is shown as 3 units (between 5 units and 2 units) which puts the midline at 3.5 units. The amplitude therefore is 1.5

In the triangle below , XZ=YZ, find the measure of< YZX

Answers

ANSWER:

80°

STEP-BY-STEP EXPLANATION:

Since the sides XZ and YZ are equal, the angle We know that the sum of all the angles is equal to 180 °, therefore:

[tex]\begin{gathered} 180=50+50+\text{YZX} \\ \text{YZX}=180-50-50 \\ \text{YZX = }80\text{\degree} \end{gathered}[/tex]

Using point P, which point would form a line parallel to line AB?
(-6,4)
(3,2)
(1,1)
(-2,3)

Answers

Answer:

(3,2)

Step-by-step explanation:

The equation of the line graphed is

y = 2/3x + 2

Parallel lines have the same slope.

y = mx + b

y = 2/3x + 0  P tells me that the y-intercept is 0

y = 2/3 x  Which point would make this true?

Point (3,2)  We plug in 3 for x and 2 for y

2 = 2/3(3)

2 = 2 This is a true statement, so the point (3,2) is on the line.

What is the simplified base of the function f(x) = (108)*?

3
33/4
633
27

Answers

Answer:33/4

Step-by-step explanation: its hard to explain but thats the correct answer 33/4 decimal form is 8.25 and mixed number is 8 1/4

Help please thank you

Answers

If $44,900 is invested at an interest rate of 7% per year,investment at the end of 5 years is

1)Annual =61712.27

2)Semiannual=62066.34

3)monthly= 62375.51

4)daily = 62287.59

What is compound interest ?

Over a specific time period, compound interest builds up on both the principal and interest. A principal is also included when accounting for interest that has accrued on it over time. The cumulative principal value is used to calculate interest for the upcoming period as well.

Given that : If $44,900 is invested at an interest rate of 7% per year .

p =44,000

R = 7%

t =5

(2)Semiannual means  compounding is in a year done means two times hence, n = 2

We know

A(t) =p[tex][1 +\frac{r}{n} ]^{nt}[/tex]

A(5) = 44000[tex][1+\frac{0.07}{2} ]^{2(5)}[/tex]

A(5)=62066.34

(1) Annual means. compounding is done 1 time for  a year

So, n = 1

A (5) = 44000[tex][1 + 0.07]^{(5)}\\[/tex]

A(5)= 61712.27

(3)Monthly means compounding is done 12 times in a year.

So, n = 12

= 44000[tex][1 + \frac{0.07}{12} ]^{60}\\[/tex]

= 62375.51

(4)For daily put n = 365

A(5) = 44000[tex][1 + \frac{0.07}{5} ]^{5(5)}\\[/tex]

= 44000[tex][1 + \frac{0.07}{5} ]^{25}\\[/tex]

=62287.59

To know learn more about compound interest visit :

brainly.com/question/14295570
#SPJ9

PLEASE HELP ME 1. A different pool had an area that is of the form
▢ × 102 + ▢ × 101 + 6
and that can be written in the form x3 ,
where x is a whole number.

A) Decide what your number could be.



B) What is the perfect square number that is closest to the number you chose? What would the side length of a square pool with that area be?


C) Estimate the side length of a square pool with the area you chose in part a).

Answers

The solutions to the questions are

The number we are using is 5The perfect square number closest to the area is 1024The estimated side length is 32

What are areas?

The area of a shape is the amount of space on the shape

For most regular quadrilaterals, you multiply the side lengths to determine the area

The number to use

The area expression is given as

Area = ▢ × 102 + ▢ × 101 + 6

From the question, we can use any number

However, this number must be positive

Assume the number is 5

So, we have

Area = 5 × 102 + 5 × 101 + 6

The perfect square number closest to the result

In (a), we have:

Area = 5 × 102 + 5 × 101 + 6

Evaluate

Area = 1021

The perfect square number closest to this is

Closest = 1024

The estimated side length

In (b), we have

Closest = 1024

Rewrite as

Area = 1024

Take the square roots

Length = 32

Read more about areas at

https://brainly.com/question/16184187

#SPJ1

Florence wishes to retire at age 65 with $1,200,000 in her retirement account. When she turns 22, she decides to begin depositing money into an account with an APR of 7% compounded monthly. What is the monthly deposit that Florence must make in order to reach her goal? Round your answer to the nearest cent, if necessary.

Answers

The monthly deposit that Florence must make in order to reach her goal is  $366.74.

What should be the monthly deposit?

The series of payments that must be made is known as an annuity. An annuity is a series of payment made over a period of time. The amount that is invested monthly would grow at an exponential rate. This is because both the amount invested and the interest that has been accrued grows at each compounding period.

The formula to determine the monthly deposit is:

Monthly deposit = future value / annuity factor

Annuity factor = {[(1+r)^n] - 1} / r

Where:

r = interest rate = 7 / 12 = 0.583%n = number of periods = number of years x number of compounding periods

(65 - 22) x 12

43 x 12 = 516

1,200,000 ÷ [((1.00583)^516]}- 1 / 0.00583] = $366.74

To learn more about annuities, please check: https://brainly.com/question/24108530

#SPJ1

In the diagram the smaller triangle is an image of the larger triangle. a) Name the angle in the image that corresponds to ZT in the preimage. b) List all pairs of corresponding sides.

Answers

this is transformations or enlagement of triangle by a certain scale factor.

which simply means it can either be double, or shortened .

solution

a. angle T is the preimange of angle T'

b. line RS and RS'

c. line RT and RT'

line ST and ST'

extra note: this is ∆RST filled over but shortened by x factors to mirror image ∆RST

graph and label each figure and its image under the translation along the given vector. identify the coordinates of the image.

Answers

The transformations are

Transformation 1: The image of the transformation are P'(-3, 0), Q'(4, 4), R'(8, -4) and S'(1, -7)Transformation 2: The image of the transformation are E'(1, -4), F'(6, 0) and G'(7, -4)

How to perform the transformation

Transformation 1

Here, we have

The coordinates are given as

P(-5, 4), Q(2, 8), R(6, 1) and S(-1, -3)

The transformation is given as

<2, -4>

This is represented as

(x, y) ⇒ (x + 2, y - 4)

This means that the translation is (x + 2, y - 4)

When this rule is applied on the coordinates, we have

P'(-3, 0), Q'(4, 4), R'(8, -4) and S'(1, -7)

Next, we plot the image of the transformation on a coordinate plane

See attachment for the image of the transformation

Transformation 2

Here, we have

The coordinates are given as

E(-7, -7), F(-2, -3), G(-1, -6)

The transformation is given as

<8, 3>

This is represented as

(x, y) ⇒ (x + 8, y + 3)

This means that the translation is (x + 8, y + 3)

When this rule is applied on the coordinates, we have

E'(1, -4), F'(6, 0) and G'(7, -4)

Next, we plot the image of the transformation on a coordinate plane

See attachment for the image of the transformation

Read more about transformation at

https://brainly.com/question/27224272

#SPJ1

Tell whether each number is a real number, a rational number, an irrational number, an integer or a whole number. Then order the numbers from these to greatest
26. -11.5, -11, -10, 12.5, 12

Answers

The order from smaller to greatest is -11.5, -11, -10, 12, 12.5.

What are whole number?

Zero, a positive natural number, or a negative integer denoted by a minus sign are all examples of integers. The inverse additives of the equivalent positive numbers are the negative numbers. The set of integers is frequently represented in mathematical notation by the boldface Z or blackboard bold math Z.

What are real numbers?

The value of a continuous quantity that can represent a distance along a line is a real number. René Descartes first used the word "real" in this meaning in the 17th century when he made a distinction between the real and imaginary roots of polynomials.

Given numbers are -11.5, -11, -10, 12.5, 12

The order from smaller to greatest is -11.5, -11, -10, 12, 12.5.

Real numbers: -11.5, -11, -10, 12, 12.5.

Rational numbers: -115/10, 125/10

Integer numbers: -11.5, -11, -10, 12, 12.5.

Whole numbers: 12, 12.5, 26

Therefore, the order from smaller to greatest is -11.5, -11, -10, 12, 12.5.

To know more about real numbers, visit:

https://brainly.com/question/551408

#SPJ9

Classify the following
sequence:
1, 4, 9, 16, 25, ...
A. neither
B. geometric
C. arithmetic

Answers

FOR ARITHMETIC

[tex]lhs = 9 - 4 \\ lhs = 5 \\ rhs = 4 - 1 \\ rhs= 3 \\ \\ lhs \: not = rhs[/tex]

SINCE THERE IS NO COMMON DIFFERENCE LET US TEST FOR A GEOMETRIC SEQUENCE.

[tex]lhs = \frac{9}{4} \\ \\ rhs = \frac{4}{1} \\ rhs = 4 \\ \\ lhs \: not = rhs[/tex]

THERE IS NO COMMON DIFFERENCE NOR A COMMON RATION THIS MEANS THAT THE SEQUENCE IS NEITHER AN ARITHMETIC NOR A GEOMETRIC SEQUENCE.

THE OPTION IS A.NEITHER.

A bag of marbles contains 4 green marbles, 3 blue marbles, 2 red marbles, and 5 yellow marbles. How many total
possible outcomes are there when choosing a marble from the bag?


Answers

Answer: 14

Step-by-step explanation: https://brainly.com/question/15694490

devin prepares for a multi day hike by packing his backpack with food,water,bedding, and emergency supplies. the tracks how many pounds of each he's carrying and find the ratio of their respective weights 2:3:1:4 f the total weight of his supplies is 35 pounds how many pounds of water is devin carrying

Answers

The amount in pounds of water that's been carried by Devin as described by the ratio given in the task content is; 10.5 pounds.

How many pounds of water is Devin carrying as described in the task content?

It follows from the task content that the amount of water Devin is carrying is to be determined.

On this note, since the backpack contains food, water, bedding, and emergency supplies are in the ratio; 2 : 3 : 1 : 4.

It therefore follows from proportion that the ratio representation of total of all supplies is; (2 + 3 + 1 + 4) = 10.

Therefore, since the total weight is; 35 pounds, it follows that the amount of water Devin is carrying can be of help as follows;

w/35 = 3/10.

w = (3 × 35)/10

w = 105/10

w = 10.5 pounds.

Ultimately, the total weight of water Devin is carrying is therefore; 10.5 pounds.

Read more on ratios;

https://brainly.com/question/2914376

#SPJ1

Hi hi job hi thank everyone for

Answers

We are asked to determine the value of the variation of the angle with respect to "x". To do that we will determine an equation that relates "x" and the angle using the following right triangle:

We can use the function tangent since this is defined as:

[tex]\tan\theta=\frac{opposite}{adjacent}[/tex]

Now, we plug in the values:

[tex]\tan\theta=\frac{3000}{x}[/tex]

Now, we will determine the angle when "x = 2000". We substitute 2000 in the equation:

[tex]\tan\theta=\frac{3000}{2000}[/tex]

Now, we take the inverse function of the tangent:

[tex]\theta=\tan^{-1}(\frac{3000}{2000})[/tex]

Solving the operations we get:

[tex]\theta=56.31°[/tex]

Therefore, the angle is 56.31 degrees or 0.983 radians.

We are asked to determine the value of the derivative of "x" with respect to "t". Since this is equivalent to the velocity of the plane we have that:

[tex]\frac{dx}{dt}=-200\frac{ft}{s}[/tex]

Now, we are asked to determine the derivative of the angle with respect to time. To do that we will determine the derivative with respect to time on both sides of the equation:

[tex]\frac{d}{dt}(\tan\theta)=\frac{d}{dt}(\frac{3000}{x})[/tex]

For the derivative on the left side, we know that the derivative of the tangent is the secant squared, therefore, we have:

[tex]\sec^2(\theta)\frac{d\theta}{dt}=\frac{d}{dt}(\frac{3000}{x})[/tex]

Now, we determine the derivative on the right side using the following formula:

[tex]\frac{d}{dx}(\frac{a}{x})=-\frac{a}{x^2}[/tex]

Applying the rule we get:

[tex]\sec^2\theta\frac{d\theta}{dt}=-\frac{3000}{x^2}\frac{dx}{dt}[/tex]

Now, we solve for the derivative of the angle with respect to time:

[tex]\frac{d\theta}{dt}=-\frac{1}{\sec^2\theta}(\frac{3000}{x^2})\frac{dx}{dt}[/tex]

Now, we substitute the values:

[tex]\frac{d\theta}{dt}=-\frac{1}{\sec^2(0.983)}(\frac{3000ft}{(2000ft)^2})(-200\frac{ft}{s})[/tex]

Solving the operations:

[tex]\frac{d\theta}{dt}=0.047\frac{rad}{s}[/tex]

Therefore, the variation of the angle with respect to time is 0.047 rad/s.

This means that the dish is rotating at a speed of 0.047 rad/s.

Answer: We are asked to determine the value of the variation of the angle with respect to "x". To do that we will determine an equation that relates "x" and the angle using the following right triangle:We can use the function tangent since this is defined as:Now, we plug in the values:Now, we will determine the angle when "x = 2000". We substitute 2000 in the equation:Now, we take the inverse function of the tangent:Solving the operations we get:Therefore, the angle is 56.31 degrees or 0.983 radians. We are asked to determine the value of the derivative of "x" with respect to "t". Since this is equivalent to the velocity of the plane we have that:Now, we are asked to determine the derivative of the angle with respect to time. To do that we will determine the derivative with respect to time on both sides of the equation:For the derivative on the left side, we know that the derivative of the tangent is the secant squared, therefore, we have:Now, we determine the derivative on the right side using the following formula:Applying the rule we get:Now, we solve for the derivative of the angle with respect to time:Now, we substitute the values:Solving the operations:Therefore, the variation of the angle with respect to time is 0.047 rad/s.This means that the dish is rotating at a speed of 0.047 rad/s.

Step-by-step explanation:

Which of the m-values satisfy the following inequality? 5m + 1 <_ 4Choose all answers that apply: A.) m = 0B.) m = 1C.) m = 2

Answers

Inequalities

We want to know which values of m satisfy ≤

5m + 1 4

Substracting 1 both sides:

-1 - 1

+03

5m ≤ 3

Dividing by 5 both sides

/5/5

m ≤ 3/5

m ≤ 0.6

Since 1 and 2 are higher than 0.6 and 0 ≤ 0.6

Then if m=0 it is satisfied

Answer: A

help me
1 Answer:

Evaluate the following expression when q = 13.
q-10
2 Answer:

Evaluate the following expression when x = 4.2.
34.5x
3 Answer:

You deposit $70 in a savings account that pays an annual interest rate
of 3%. How much simple interest would you earn in 2.5 years?

Answers

1) When q = 13, the expression q - 10 is; 3

2)  When x = 4.2, the expression 34.5x is; 144.9

3) The simple interest you would earn in 2.5 years is;  $5.25

How to solve Algebra Word problems?

1) We want to evaluate the following expression;

q - 10 when q = 13.

Thu, we will plug in 13 for q to get;

13 - 10 = 3

2) We want to evaluate the following expression;

34.5x when x = 4.2

Plug in 4.2 for x to get;

34.5 * 4.2 = 144.9

3) The formula for simple interest is;

I = PRT/100

Where;

P is principal

R is rate

T is time

We are given;

P = $70

R = 3% = 0.03

Time; T = 2.5 years

Thus;

I = 70 * 0.03 * 2.5

I = $5.25

Read more about Algebra Word problems at; https://brainly.com/question/13818690

#SPJ1

Can you solve this for me please.

Answers

a^-8 * a^3*2
=a^-8 * a^6
=a*(-8+6)
=a^-2
The answer is 1/a^2.


First, simplify the expression by multiplying exponents then calculate the product.
Other Questions
I need help with this Graphs of Quadratic Functions question. PLS HELP ME DUE TODAY what is the area of isoceles triangle What is the value of -3 + |-17|? Which of the following is a sustainable method of human societal development?A. Limiting unnecessary resource useB. Using only nonrenewable resourcesC. Degrading resources with pollutionD. Encouraging a throwaway culture Which of the following were resolutions passed by Federalist leaders at the Hartford Convention in December 1814?a. Require a two-thirds majority in Congress in order to declare war.b. Abolish the three-fifths rule.c. Sign an immediate peace treaty with Great Britain.d. New England would secede from the Union. Answer this please please Common methods of securing an IT system do NOT include A) protecting system integrity by using antimalware programs B) protecting data integrity by unblocking unauthorized access C) protecting data integrity by creating backups, entering and editing data correctly D) setting permissions to restrict access Name Line / Line Segment / RayJul 26, 6:43:31 PM?Name the figure below in two different ways.DSymbol:andO Why did President Wilson's decision to stay out of World War I in 1914 please many Americans? the process where an external qualitative methodology or topic area specialists (such as subject matter experts) are asked to review the research analysis is known as a the number of representatives (called for in the constitution of 1787) appears to be too few, either to communicate the requisite (required) information of the wants, local circumstances, and sentiments of so extensive an empire, or to prevent corruption and undue influences in exigencies (urgent need/demand) of such great powers Juliet brought 506 strawberries and watermelon. A few day later, she found that 1/3 or the strawberries and 10 watermelon were rotten. The remaining number of strawberries and watermelon were in the ratio of 2:1. How many strawberries were rotten? a 22-year-old client is admitted to room 13. he states that he does not want to remain in the room because the number will bring him bad luck. the nurse should: ____________ is a way to turn a company into a parent company with smaller retail outlets owned by independent operators.A.ManufacturingB.IncorporatingC.TaxingD.Franchising passage.............. Which of these scenarios is an example of social pressure that influences behavior Contrast the medieval attitude you described above with European attitudes that emerged during the Modern period (after 1492). What was the last armed conflict to take place between the native Americans and the US army Chapter 1: BeginningsTHE CONSTITUTIONUse pages 12-14 of the Student Text to complete the worksheet.SECTION 5MATCHINGMatch each word to its definition, and write the letter in the blank.1. popular sovereignty2. judicial review3. federalism4. unconstitutional5. vetoA. actions or laws not allowed bythe ConstitutionB. the power of the courts to decideif laws and other governmentactions are unconstitutionalC. the idea that the people are thesupreme rulers of the governmentD. to reject and send backE. the national government and thestate governments share power